问答

4答疑

[Sentence Correction]

试题详情

题目:

Before 1988, insurance companies in California were free to charge whatever rates the market would bear, needing no approval from regulators before raising rates.

选项:

答案:

A
4答疑

[Critical Reasoning]

试题详情

题目:

The higher the level of certain vitamins and minerals in the bloodstream, the better a person's lung function, as measured by the amount of air the person can expel in one second.  The lung function of smokers is significantly worse, on average, than that of nonsmokers.  Clearly, therefore, one way for smokers to improve their lung function is for them to increase their intake of foods that are rich in these helpful vitamins and minerals.
 
Which of the following is an assumption on which this argument depends?

选项:

A、Smokers are less likely than nonsmokers to have diets that are rich in vitamins and minerals.
B、The lung function of smokers whose diets are rich in those vitamins and minerals is generally better than that of nonsmokers with comparable diets.
C、People whose diets are deficient in those vitamins and minerals do not typically have other health problems in addition to diminished lung function.
D、Stopping smoking will not typically improve lung function more than any diet changes can.
E、Smoking does not introduce into the body chemicals that prevent the helpful vitamins and minerals from entering the bloodstream.

答案:

E
4答疑

[Sentence Correction]

试题详情

题目:

Even though sub-Saharan Africa often evokes images of drought and famine, researchers say that the area is the home of more than 2,000 grains, vegetables, roots, fruits, and other foods that have the possibility for feeding the continent, even other parts of the world.

选项:

A、 have the possibility for feeding the continent, even
B、 have the possibility of feeding the continent, even of
C、 could feed the continent, so even with
D、 could feed the continent and even
E、 could possibly feed the continent and, in addition, for even

答案:

D
4答疑

[Sentence Correction]

试题详情

题目:

Severely hindered by problems with local suppliers, the fact that the AQ division also had a new management team to adapt to was not seen by the board of directors as a legitimate excuse for such low productivity.

选项:

A、Severely hindered by problems with local suppliers, the fact that the AQ division also had a new management team to adapt to
B、Though severely hindered by local supply problems, the fact that the AQ division also had a new management team to which to adapt
C、Severely hindered by problems with local suppliers, the AQ division also had to adapt to a new management team, but this
D、Severely hindered by local supply problems, that the AQ division also had to adapt to a new management team
E、Though severely hindered by problems with local suppliers, the AQ division's also having a new management team to which it had to adapt

答案:

C
4答疑

[Sentence Correction]

试题详情

题目:

In an attempt to attract more tourists, the Swiss government commissioned several life-size fiberglass cow statues, having them decorated by local artists, and set them up on the streets of Zurich.

选项:

A、the Swiss government commissioned several life-size fiberglass cow statues, having them decorated by local artists, and
B、the Swiss government commissioned several life-size fiberglass cow statues, they were decorated by local artists, and then
C、the Swiss government commissioned several life-size fiberglass cow statues, had them decorated by local artists, and
D、several life-size fiberglass cow statues were commissioned by the Swiss government, decorated by local artists, and then
E、several life-size fiberglass cow statues, commissioned by the Swiss government, were decorated by local artists, and they

答案:

C
4答疑

[Data Sufficiency]

试题详情

题目:

If m and n are positive integers, what is the value of
  1. mn = 12
  2. is in lowest terms and is in lowest terms.

选项:

A、Statement (1) ALONE is sufficient, but statement (2) alone is not sufficient.
B、Statement (2) ALONE is sufficient, but statement (1) alone is not sufficient.
C、BOTH statements TOGETHER are sufficient, but NEITHER statement ALONE is sufficient.
D、EACH statement ALONE is sufficient.
E、Statements (1) and (2) TOGETHER are NOT sufficient.

答案:

C
4答疑

[Sentence Correction]

试题详情

题目:

With a new park, stadium, and entertainment complex along the Delaware River, Trenton, New Jersey, is but one of a large number of communities that is looking to use its waterfront as a way for it to improve the quality of urban life and attract new businesses.

选项:

A、is looking to use its waterfront as a way for it to improve the quality of urban life and attract
B、is looking at using its waterfront to improve the quality of urban life and attract
C、are looking to use their waterfronts to improve the quality of urban life and attract
D、are looking to use its waterfront as a way of improving the quality of urban life and attracting
E、are looking at using their waterfronts as a way of improving the quality of urban life and attract

答案:

C
4答疑

[Critical Reasoning]

试题详情

题目:

Economist:  On average, the emergency treatment for an elderly person for injuries resulting from a fall costs $11,000.  A new therapeutic program can significantly reduce an elderly person's chances of falling.  Though obviously desirable for many reasons, this treatment program will cost $12,500 and thus cannot be justified.
 
Which of the following, if true, most seriously undermines the conclusion of the argument?

选项:

A、Among elderly people who had followed the program for only a few months, the number of serious falls reported was higher than it was for people who had followed the program for its recommended minimum length of one year.
B、Falls resulting in serious injuries are less common among elderly people living in nursing homes than they are among elderly people who live alone at home.
C、A frequent result of injuries sustained in falls is long-term pain, medication for which is not counted among the average per-person costs of emergency treatment for elderly people's injuries from such falls.
D、The new therapeutic program focuses on therapies other than medication, since overmedication can cause disorientation and hence increase the likelihood that an elderly person will have a serious fall.
E、A significant portion of the cost of the new therapeutic program is represented by regular visits by health care professionals, the costs of which tend to increase more rapidly than do those of other elements of the program.

答案:

C
4答疑

[Sentence Correction]

试题详情

题目:

The recent decline in the value of the dollar was triggered by a prediction of slower economic growth in the coming year.  But that prediction would not have adversely affected the dollar had it not been for the government's huge budget deficit, which must therefore be decreased to prevent future currency declines.
Which of the following, if true, would most seriously weaken the conclusion about how to prevent future currency declines?

选项:

A、The government has made little attempt to reduce the budget deficit.
B、The budget deficit has not caused a slowdown in economic growth.
C、The value of the dollar declined several times in the year prior to the recent prediction of slower economic growth.
D、Before there was a large budget deficit, predictions of slower economic growth frequently caused declines in the dollar's value.
E、When there is a large budget deficit, other events in addition to predictions of slower economic growth sometimes trigger declines in currency value.

答案:

D
4答疑

[Problem Solving]

试题详情

题目:

The probability that event M will not occur is 0.8 and the probability that event R will not occur is 0.6. If events M and R cannot both occur, which of the following is the probability that either event M or event R will occur?

选项:

A、
B、
C、
D、
E、

答案:

C
0评分
400浏览

[Undefined]

An unusually severe winter occurred in Europe after the continent was blanketed by a blue haze resulting from the eruption of the Laki Volcano in the European republic of Iceland in the summer of 1984.  Thus, it is evident that major eruptions cause the atmosphere to become cooler than it would be otherwise.
Which of the following statements, if true, most seriously weakens the argument above?
C选项为什么就不会有张冠李戴的问题呢,1984和1982的气候情况能一样吗,C里的火山就可以试用于原文的火山吗
0评分
447浏览

[Undefined]

The rate of a certain chemical reaction is directly proportional to the square of the concentration of chemical A present and inversely proportional to the concentration of chemical B present. If the concentration of chemical B is increased by 100 percent, which of the following is closest to the percent change in the concentration of chemical A required to keep the reaction rate unchanged?
为什么这个题一定是相乘不能是相加呢?y=k1A方-k2B (k1,k2大于0 ),也满足题目说的与A方成正比与B成反比啊?但是这道题只有相乘的情况才能选出正确答案
0评分
323浏览

[Undefined]

Tiger beetles are such fast runners that they can capture virtually any nonflying insect.  However, when running toward an insect, the beetles intermittently stop, and then, a moment later, resume their attack.  Perhaps they cannot maintain their pace and must pause for a moment's rest; but an alternative hypothesis is that while running tiger beetles are unable to process the resulting rapidly changing visual information, and so quickly go blind and stop.
 
Which of the following, if discovered in experiments using artificially moved prey insects, would support one of the two hypotheses and undermine the other?
答案C是怎么支持一个猜想又削弱另一个猜想的?
0评分
705浏览

[GWD]

The author of the passage implies that which of the following is a possible partial explanation for acquisition behavior during the 1970's and 1980's?
请解释这道题为什么不选C或E,除了正确选项B,我认为文中也正确地提到了这两个选项可以解释acquisition behavior during 1970s and 80s
0评分
612浏览

[OG]

The primary purpose of the passage is to
为什么选E不选B
0评分
405浏览

[Undefined]

Club X has more than 10 but fewer than 40 members. Sometimes the members sit at tables with 3 members at one table and 4 members at each of the other tables, and sometimes they sit at tables with 3 members at one table and 5 members at each of the other tables. If they sit at tables with 6 members at each table except one and fewer than 6 members at that one table, how many members will be at the table that has fewer than 6 members?
提问小梅老师 这道题求讲一下 我没什么思路 带入数字得的是3
0评分
415浏览

[Undefined]

A photography dealer ordered 60 Model X cameras to be sold for $250 each, which represents a 20 percent markup over the dealer's initial cost for each camera. Of the cameras ordered, 6 were never sold and were returned to the manufacturer for a refund of 50 percent of the dealer's initial cost. What was the dealer's approximate profit or loss as a percent of the dealer's initial cost for the 60 cameras?
请问小梅老师 这道题讲讲一下 我算的是19% 原来成本是200每个 亏了6个 每个亏100. 最后得不出答案
0评分
90888浏览

[Undefined]

Analytical!
请问may老师,这题该用什么思路来做?
0评分
97969浏览

[Undefined]

Analytical!
请问张老师,这道题为什么D不对?我有我觉得second paragraph最后一句话特别指向D
1评分
99932浏览

[Undefined]

Analytical!
请问老师,这道题为什么second part是”evidence”? 还有、不确定First part到底是background info还是premise?
点我领取
免费专项课程
在线咨询